LSAT and Law School Admissions Forum

Get expert LSAT preparation and law school admissions advice from PowerScore Test Preparation.

User avatar
 Dave Killoran
PowerScore Staff
  • PowerScore Staff
  • Posts: 5852
  • Joined: Mar 25, 2011
|
#92490
Complete Question Explanation

Justify the Conclusion. The correct answer choice is (E)

The argument is given in premise-conclusion-premise form, and the conclusion is that "the city needs to purchase new subway cars." The word "so" often introduces a conclusion, and the word "since" often introduces a premise. The argument itself is weak since there may be other ways to have an efficient attractive subway system other than by purchasing new subways cars (for instance, the city could renovate the existing system).

The question stem asks you to find the statement that, when combined with the premises, will make the conclusion a logical conclusion. From this perspective, when the correct answer choice is added to the premises, the conclusion should automatically follow. It is therefore important to establish that the correct answer choice will produce the conclusion that new subway cars need to be purchased by the city.

Answer Choice (A): Incorrect. This answer choice suggests that the city should do something to improve the subway system, but it does not specify that the city must purchase new subway cars.

Answer Choice (B): Incorrect. This answer is incorrect because it addresses "cost-effective subway cars," whereas the conclusion discusses "new subway cars."

Answer Choice (C): Incorrect. The conclusion is clear that the city "needs" to purchase new subway cars. Answer choice (C) simply states that subway cars make better economic sense than "many of the other investment options open to the city"; this does not produce the conclusion that the city must purchase new subway cars but rather that purchasing such cars was a reasonable decision.

Answer Choice (D): Incorrect. Affordability is not the issue in the stimulus.

Answer Choice (E): This is the correct answer choice. This answer choice can be diagrammed using conditional arrows, as follows:

  • Efficient and attractive subway system :arrow: New subway cars

According to the premises in the stimulus, having an efficient and attractive subway system makes good economic sense, and the city always does what makes good economic sense. Therefore, the city will strive to have an efficient and attractive subway system. Combining this with answer choice (E), it follows that the city needs new subway cars. Since this matches the conclusion in the stimulus, answer choice (E) must be correct.
 Adam Tyson
PowerScore Staff
  • PowerScore Staff
  • Posts: 5153
  • Joined: Apr 14, 2011
|
#43569
This question came to us in another thread in the forum:
Hello!
I am having trouble with the logic of this very simple question. I can answer it no problem with intuition, but when I write out the logic, the answer is unclear.

It has 2 premises
P1: Efficient and attractive subway systems make good economic sense.
P2: The city always does what makes good economic sense.
Conclusion: The city needs to buy new subway cars.

Obviously, the gap is between buying new subway cars and efficient and attractive subway systems.
I first diagrammed the logic like this: EAS-->GES C--->GES thus PNSC-->EAS--GEC. So I was looking for if you purchase new cars then you get efficient/attractive subways. This is a mistaken reversal. But how do you get purchasing new cars in the necessary condition?

Thank you, anyone!
Thanks for the question! Part of the problem you may be experiencing is that tried to force too much into conditional terms and diagrams. For example, the first sentence about making good economic sense has no conditional aspects to it, in my opinion. There is no "if this, then that" relationship - it's just "this is good". With the second premise, you have treated a "should" claim as a conditional claim. "Should" is not a conditional indicator!

"Needs", however, IS a necessary condition indicator, and our author told us in his conclusion that the city needs to buy new subway cars. That means he thinks they are a necessary condition. Our goal in order to justify this claim is to make the premises sufficient for the conclusion. If it's true that the city should do whatever makes good, economic sense, and if an efficient, attractive subway system does that, then clearly the city should make the system efficient and attractive. But how do we make the leap from there to buying new cars? Make the efficient, attractive system (the premises) sufficient for the new cars (the conclusion). That's how I would approach it. Rather than trying to make all the premises conditional within themselves, just make the premises as a whole a sufficient condition for your conclusion. Make subway cars necessary!
 Sandy180
  • Posts: 2
  • Joined: Feb 04, 2018
|
#43659
Wow! That question was driving me nuts. Amazing explanation and what a relief. I totally understand. I am still not comfortable with conditional logic in LR, so I am looking for every opportunity to put it into practices. Very happy so far with the LR Bible. I might have to read it twice, though.

Anyway, big help! Thank you

Get the most out of your LSAT Prep Plus subscription.

Analyze and track your performance with our Testing and Analytics Package.